Glamorous Geometric Series

Integers a a , b b , c c , d d and e e satisfy 50 < a < b < c < d < e < 500 50 < a < b < c < d < e < 500 , and a , b , c , d , e a,b,c,d,e form a geometric sequence. What is the sum of all possible distinct values of a a ?


The answer is 321.

This section requires Javascript.
You are seeing this because something didn't load right. We suggest you, (a) try refreshing the page, (b) enabling javascript if it is disabled on your browser and, finally, (c) loading the non-javascript version of this page . We're sorry about the hassle.

11 solutions

Andrew Kwon
May 20, 2014

Because all of the terms in the geometric sequence are integers, the common ratio must be rational. Thus, we can write the sequence as a , a ( m n ) , a ( m n ) 2 , a ( m n ) 3 , a ( m n ) 4 a, a\left(\frac{m}{n} \right), a \left(\frac{m}{n} \right)^2, a \left(\frac{m}{n} \right)^3, a \left(\frac{m}{n} \right)^4 , where m , n m,n are relatively prime, and m > n m>n (because the sequence must be strictly increasing). Now, the most important property of a a is that n 4 n^4 must divide it. Noting that n = 1 n=1 does not lend itself to any possible values for a a , we begin our casework with n = 2 n=2 .

If n = 2 n=2 , then 16 a 16 \mid a , and a > 50 a>50 . Thus, the smallest possible value for a a is 64 64 . We need only to confirm that there exists an integer m m such that m > n m>n and that 64 2 4 m 4 = 4 m 4 < 500 \frac{64}{2^4} \cdot m^4=4m^4<500 . Indeed, m = 3 m=3 works. and so a = 64 a=64 is a solution. We continue this process for n = 2 n=2 , finding a = 80 , 96 a=80,96 are also possible starting values. However, if a > = 112 a>=112 , we find that m m must be an integer less than 3 3 , and so there are no more solutions for this case.

Continuing with n = 3 n=3 in a similar fashion, we find a = 81 a=81 to be the smallest multiple of 81 81 greater than 50 50 . Easily, m = 4 m=4 is the only value of m m that satisfies all conditions. Again, note that for a > = 162 a>=162 , m m must be an integer less than 4 4 , which does not satisfy m > n m>n , and so there are no more solutions.

Considering n = 4 n=4 , it follows that the smallest possible value for m m is 5 5 ; however, this yields that e = 625 a n 4 e=625 \cdot \frac{a}{n^4} (with a n 4 \frac{a}{n^4} being an integer), which clearly is greater than 500 500 . Thus, there are no more solutions.

From this casework, the solutions are a = 64 , 80 , 96 , 81 a=64, 80, 96, 81 , which sum to 321 \boxed{321} .

This is a pretty well written solution. There is slight issue here: the last case should be n 4 , n\geq 4, not n = 4. n=4. The most common inaccuracy was failing to rigorously justify that n 4 n^4 must divide a . a.

Calvin Lin Staff - 7 years ago

Exactly Same Way.

Kushagra Sahni - 5 years, 4 months ago
Siam Habib
May 20, 2014

Since, it is a geometric sequence, there is a common ratio between the consecutive terms of the sequence. Lets assume the common ratio is r r .

Firstly, we will prove that 1 < r < 2 1 < r < 2 .

Since, a < b a < b , it is apparent that r = b a > 1 r = \frac{b}{a} > 1 .

r < 2 r<2 because, if r 2 r \geq 2 , then, the minimum possible value of the fifth term of the sequence a r 4 = 50 × 2 5 = 800 > 500 ar^4 = 50 \times 2^5 = 800 > 500 which contradicts with the given conditions.

So, 1 < r < 2 1 < r < 2 .

It is also apparent that r r is rational number, because, a , b , c , d , e ϵ N a,b,c,d,e \epsilon N and the product of a irrational number and a natural number is always irrational.

So, we can write that r = x y r = \frac{x}{y} where x , y ϵ N x,y \epsilon N and they are co-prime and x > y x > y .

Now, for a , b , c , d a,b,c,d and e e to be integers, a a has to be divisible by y 4 y^4 . Otherwise, a r 4 = a x 4 y 4 ar^4 = a\frac{x^4}{y^4} would not be an integer.

Since, a a is divisible by y 4 y^4 , we can see that the minimum value of a y 4 = 1 \frac {a}{y^4} = 1 and therefore the minimum value of a r 4 = a × x 4 y 4 = x 4 ar^4 = a \times \frac{x^4}{y^4} = x^4 .
And since, the value of e = a r 4 e = ar^4 cannot exceed 500 500 , therefore x 4 < 500 x^4 < 500 and so x < 500 4 4.7 x < \sqrt[4]{500} \approx 4.7 .So, the maximum possible value of x = 4 x = 4 .

We can also see that the minimum value of x = 3 x =3 . Cause, we said earlier that x > y x>y and x , y x,y are positive integers.If x 2 x\leq2 , then the only possible values of x x are 2 2 and 1 1 . [Since, x x is a positive integer.]
But, we said that x > y x>y . But, there is no positive integer less than 1. And since y y has to be a positive integer, so there is no possible value of y y when x = 1 x =1 .Similarly, when x = 2 x=2 , y = 1 y=1 .But, then, r = 2 r = 2 . But, we have proved that 1 < r < 2 1< r <2 So, x > 2 x > 2 . So, the minimum value of x x is 3 3 .

So, we can say that 3 x 4 3 \leq x \leq 4 .

So, the only possible values of x x are 4 4 and 3 3 .

Now, when x = 4 x = 4 , 3 3 is the only possible value for y y .
[ Since, y > 1 y > 1 and y < x y<x and x x and y y are co-prime.]
So, r = 4 3 r = \frac {4}{3} .
So, the minimum value of a = 3 4 a = 3^4 . This is the only value of a a . Because, since a a is divisible by 3 4 3^4 , we can write a = k × 3 4 a = k \times 3^4 where k ϵ N k \epsilon N .
Then a r 4 = k × 4 4 < 500 ar^4 = k \times 4^4 < 500
or, k < 500 4 4 k < \frac{500}{4^4}
or, k < 1 61 64 k < 1\frac{61}{64}
Since, k k is a natural number, the only possible value of k k is 1.

In the same way, When x = 3 x = 3 , the only value of y y is 2 2 .
So, the only value of r r is 3 2 \frac{3}{2} .
Since, a a is divisible by 2 4 2^4 , we can write that a = k × 2 4 a = k \times 2^4 where k ϵ N k \epsilon N .
So, e = a r 4 = 2 4 × k × 3 4 2 4 = k × 3 4 e = ar^4 = 2^4 \times k \times \frac {3^4}{2^4} =k \times 3^4
So, k × 3 4 < 500 k \times 3^4 < 500
or, k < 500 3 4 k < \frac{500}{3^4}
or, k < 6 14 81 k < 6\frac{14}{81}
So, the maximum value of k k is 6 6 .
On the other hand, k × 2 4 > 50 k \times 2^4 >50
or, k > 3 1 8 k > 3\frac{1}{8}
So, the minimum value of k k is 4 4 .
So, the sum of all the possible values of a a in this case is k = 4 6 ( k × 2 4 ) \displaystyle \sum_{k=4}^6 {(k \times 2^4}) .

So, the sum of all the possible values of a = 3 4 + k = 4 6 ( k × 2 4 ) = 15 × 2 4 + 3 4 = 321 a = 3^4 +\displaystyle \sum_{k=4}^6 {(k \times 2^4}) = 15 \times 2^4 +3^4 = 321 .
So, the answer is 321 321 .

Very detailed solution

Calvin Lin Staff - 7 years ago
Shourya Pandey
May 20, 2014

Suppose the common ratio is a positive integer, say x x ,where x 1 x \neq 1 , or that x 2 x \geq 2 . Then e = a x 4 > a 2 4 = 16 a > 800 > e e=a*x^{4} > a*2^{4}=16a > 800 >e , a contradiction.

So the common ratio x x is not an integer. However it is surely a rational number, because x = b a x = \frac {b}{a} . Let x = p q x= \frac {p}{q} in its reduced form, where q 1 q \neq 1 and p > q p>q , since the G.P. is increasing.

So e = a p 4 q 4 e= \frac {a*p^{4}}{q^{4}} , so that q 4 q^{4} must be a factor of a a . Suppose q 4 q \geq 4 , then clearly,

p q q + 1 q ( 1 + 1 q ) 5 4 \frac {p}{q} \geq \frac {q+1}{q} \geq (1+\frac {1}{q}) \geq \frac {5}{4} . Also a q 4 256 a \geq q^{4} \geq 256

Thus e 256 ( 5 4 ) 4 625 e \geq 256* (\frac {5}{4})^{4} \geq 625 , a contradiction.

Thus q = 2 , 3 q=2,3 . If q = 2 q= 2 , then p = 3 p=3 is the only possibility, otherwise p q \frac {p}{q} will be 2 2 or more. So x = 3 2 x=\frac {3}{2} , and 16 16 is a factor of a a . Clearly e = 81 a 16 < 500 e = \frac {81a}{16} < 500 , or a < 99 a<99 , and thus a = 64 , 80 , 96 a=64,80,96 .

If q = 3 q=3 , then a 81 a \geq 81 and p = 4 , 5 p=4,5 . But for p = 5 p=5 , e = a x 4 81 5 4 3 4 = 625 e e = a*x^{4} \geq \frac {81*5^4}{3^4} = 625 \geq e , a contradiction. So p = 4 p=4 . Now e = 256 a 81 < 500 e= \frac {256a}{81} < 500 , or a < 159 a< 159 . But since 81 81 is a factor of a a , a = 81 a=81 .

Thus the sum of all possible values of a a are 64 + 80 + 96 + 81 = 321 64+80+96+81 = 321 .

Owen Scott
May 20, 2014

Given: geometric series of integers increasing between 50 and 500. Terms (a,b,c,d,e) could be written as (a, a r, a r^2, a r^3, a r^4) with r being the ratio between terms in the geometric series.

r must be greater than 1 for it to be an increasing geometric series. r^4 must be less than 10 for it to be between 50 and 500 (which have a ratio of 10)

Roughly, this means that r^2 < 3 and r < 1.75. In summary, 1 < r < 1.75

In order for a,b,c,d,e to all be integers, the ratio must be a rational number. Let us call this ratio x/y a must be evenly divisible by y^4 or not all a,b,c,d,e will be integers

Now, we must remember that a * (x/y)^4 must be less than 500. If x is 5 or greater, we will be multiplying by 625 in the numerator. a/(y^4) will be one or greater or else not all of a,b,c,d,e will be integers. This means that x must be smaller than 5.

If x=2, then the only possible ratio greater than 1 is 2/1, which does not satisfy r^4 < 10.

Thus, x=3 or x=4.

If x=3, y=2 (if y=1, r^4 is not less than 10). Thus, a must be a power of 16 (so that a is evenly divisible by y^4), and it must be greater than 50. a = 64, 80, 96 all satisfy this with a*r^4 < 500

If x=4, y=3 (if y=2, r^4 is not less than 10). Thus, a must be a power of 81 (so that a is evenly divisible by y^4). a = 81 satisfies a*r^4 < 500

64+80+96+81 = 321

Daniel Chiu
Nov 3, 2013

We have that e = a r 4 e=ar^4 , where r r is the common ratio. Since 2 4 50 = 800 > 500 2^4\cdot 50=800>500 , r < 2 r<2 . Also, a a must be divisible by a fourth power, since r r is not an integer. a a can only be divisible by 16 , 81 , 256 16,81,256 , since all other fourth powers are too big. Similarly, the numerator of r r can only be 2 , 3 , 4 2,3,4 . This gives two values of r r : 3 2 \dfrac{3}{2} and 4 3 \dfrac{4}{3} .

If r = 3 2 r=\dfrac{3}{2} , 16 a 16|a . If a 112 a\ge 112 , e 567 e\ge 567 , so a = 64 , 80 , 96 a=64,80,96 .

If r = 4 3 r=\dfrac{4}{3} , 81 a 81|a . If a 162 a\ge 162 , e 512 e\ge 512 , so a = 81 a=81 .

The answer is 64 + 80 + 96 + 81 = 321 64+80+96+81=\boxed{321} .

Nice one, I've noticed it so helpful!

Theodorus Jonathan - 7 years, 7 months ago

Let the terms be a a , a r ar , a r 2 ar^2 , a r 3 ar^3 & a r 4 ar^4 . [where r r is the common ratio with r > 1 r>1 ]

So, a > 50 a > 50 and a r 4 < 500 r < 2 ar^4 < 500 \Rightarrow r < 2

So, r r is a rational of form p q \frac{p}{q} with 1 < q < p < 2 q 1 < q < p < 2q . [ p , q p,q integers]

So, last term e = a r 4 = a p 4 q 4 e = ar^4 = \frac{ap^4}{q^4}

a = k q 4 > 50 \Rightarrow a=kq^4>50 [all terms are integers], so 2 q 3 2 \leq q \leq 3 [ q 3 q \leq 3 as q < p < 5 q < p < 5 as stated below]

Also, e = k p 4 < 500 e = kp^4 < 500 with k 1 k \geq 1 . So 3 p 4 3 \leq p \leq 4

With q = 2 , p = 3 q = 2, p = 3 , we have 3 < k < 7 3 < k < 7 giving 3 solutions with corresponding a's : 4 2 4 , 5 2 4 , 6 2 4 4*2^4, 5*2^4, 6*2^4 .

With q = 3 , p = 4 q = 3, p = 4 , we have 0 < k < 2 0 < k < 2 giving 1 solution with corresponding a: 3 4 3^4 .

So required sum is 321 321 .

Awkwardly written. Coprimality of p p and q q is not formally used. But the problem is solved

Calvin Lin Staff - 7 years ago
Jeremy Kong
May 20, 2014

Rewrite the form of the sequence terms in terms of a a and r r , the common ratio of the geometric sequence. Thus 50 < a < a r < a r 2 < a r 3 < a r 4 < 500 50 < a < ar < ar^2 < ar^3 < ar^4 < 500 . Clearly, for all of the terms to remain integers, a a must have as a factor the fourth power of some integer, say f f .

f f can either be 2, 3, or 4 since 2 4 = 16 2^4 = 16 , 3 4 = 81 3^4 = 81 and 4 4 = 256 4^4 = 256 ; note that f = 5 f = 5 will cause a = 625 > 500 a = 625 > 500 . The common ratio r r is then a fraction f + 1 f \frac{f+1}{f} - it is sufficient to consider this as we are only concerned with which values of a a satisfy the condition and we don't want a r 4 ar^4 to exceed 500.

For f = 2 f = 2 , it becomes apparent that the valid solutions of a a are 4 × 2 4 4 \times 2^4 , 5 × 2 4 5 \times 2^4 and 6 × 2 4 6 \times 2^4 , as larger constant factors will cause a r 4 > 500 ar^4 > 500 . For f = 3 f = 3 , a = 3 4 a = 3^4 is the only possible solution as larger constant factors will also cause a r 4 > 500 ar^4 > 500 . Finally for f = 4 f = 4 , we cannot get any solutions for a a as we will always have a r 4 > 500 ar^4 > 500 .

Thus our solutions for a a are 64 64 , 80 80 , 96 96 and 81 81 . Adding these up gives the desired answer 321 321 .

" a a must have as a factor the fourth power of some integer, say f f ." Not exactly rigorous.

Calvin Lin Staff - 7 years ago
Michael Tong
Nov 4, 2013

First, 1 < r < 2 1 < r < 2 since the sequence is increasing and 2 4 = 16 > 10 2^4 = 16 > 10 . Thus, r r must be a fractional number. This requires that a a have at least four of some prime factor.

Say a a has four (or more) factors of 2. The smallest such that a > 50 a > 50 is 16 4 16 * 4 . The lowest r r with 2 2 in the denominator is 3 2 \frac{3}{2} , and 64 ( 3 2 ) 4 64 * (\frac {3}{2})^4 is 4 3 4 = 324 4 * 3^4 = 324 . Thus, 64 64 works, as well as 80 80 and 96 96 , but not 112 112 .

Repeating with 3 3 , we find that only 81 81 works.

We can confirm that 4 4 and all further factors do not work for a a since, at minimum, e = a r 4 e = x 4 ( x + 1 x ) 4 e = ( x + 1 ) 4 e = ar^4 \rightarrow e = x^4 (\frac {x+1}{x})^4 \rightarrow e = (x + 1)^4 , and ( x + 1 ) 4 > 500 (x+1)^4 > 500 for all x 4 x \geq 4 .

64 + 80 + 96 + 81 = 321 64 + 80 + 96 + 81 = 321 .

Arpit Singh
May 20, 2014

As a,b,c,d,e are integers and 1<r<2

Only for r= 3/2 and 4/3 we get all integer values for a b c d e.

a= 64+80+96+81= 321

all details are missing

Calvin Lin Staff - 7 years ago

Let a = k , b = k r , c = k r 2 , d = k r 3 a = k, b = kr, c = kr^2, d=kr^3 and e = k r 4 e = kr^4 where r r is the common ratio.

r r can't be 1 1 , since a < b < c < d < e a < b < c < d < e .

Also r r can't be 2 2 , since e < 500 e < 500 .

Hence 1 < r < 2 1<r<2 . Since a , b , c , d a,b,c,d and e e are integers, the possible common ratio's are : r = 4 3 r =\frac{4}{3} and 3 2 \frac{3}{2} . For r = 4 3 a = 81 r =\frac{4}{3}\, a=81 and for r = 3 2 , a = 64 r =\frac{3}{2}, a = 64 or 80 80 or 96 96 .

Answer = 81 + 64 + 80 + 96 = 321 = 81 + 64 + 80 + 96 =\boxed{321} .

e = a * r^4

also e/a upper limit is 10 so r^4 < 10 implies r < 2

if r= 3/2, e/a = 81/16 min value of a = 64 ... for which e = 324 ... under the allowable limit also, a = 80 ... for which e = 405 ... is under the allowable limit if a = 96 ... for which e = 486 But if a = 112 ... e>500

if r= 4/3 ... e/a = 256/81 min value of a = 81 ... for which e = 256 there is no other multiple of 81 which keeps e<500

if r = 5/3 ... e/a = 625/81 ... that itself takes e>500 ... so we cannot have 5 or any other larger number in the numerator of 'r'

thus the only possible values of 'a' are: 64, 80, 96, 81 ... sum 321

0 pending reports

×

Problem Loading...

Note Loading...

Set Loading...